5x+4=x+44 solve for x

Answers

Answer 1

5x + 4 = x + 44

x is adding on the right, then it will subtract on the left

4 is adding on the left, then it will subtract on the right

5x - x = 44 - 4

4x = 40

4 is multiplying on the left, then it will divide on the right

x = 40/4

x = 10


Related Questions

Answer the question below. Be sure to show your work

Answers

ANSWER:

We have to find new side-lengths of PQR triangle.

Original sides are.

[tex]\begin{gathered} PQ\text{ = 8cm} \\ QR=17\operatorname{cm} \\ RP=15\operatorname{cm} \end{gathered}[/tex]

After multiplying by 2.5 we get

[tex]\begin{gathered} PQ^{\prime}=2.5\times PQ=(8\times2.5)cm=20\operatorname{cm} \\ QR^{\prime}=(17\times2.5)cm=42.5\operatorname{cm} \\ RP^{\prime}=(15\times2.5)cm=37.5\operatorname{cm} \end{gathered}[/tex]

These are the new sides of the P'Q'R' triangle.

Determine the coordinates of the midpoint of the segment with given endpoints. J(-3, 2), K(7,10) Midpoint:

Answers

[tex]\begin{gathered} \text{mid point = (}\frac{x_1+x_2}{2},\frac{y_1+y_2}{2}) \\ \text{mid point = (}\frac{-3+7}{2},\frac{2+10}{2}\text{)} \\ \text{mid point=(}\frac{4}{2},\frac{12}{2}\text{)} \\ \text{mid point=(2,6)} \end{gathered}[/tex]

drawing and explanation for areaof triangle where h=137 and base = 203

Answers

The area of triangle is 13905.5 cm square when height is 137 cm and bae is 203 cm.

Given that,

There is a triangle with height 137 cm and base 203 cm.

We have to find the area of triangle.

We know,

The entire area filled by a triangle's three sides in a two-dimensional plane is referred to as the triangle's area. A straightforward formula can be used to get the area of a triangle by multiplying the sum of the base and height by two.

Area of triangle =1/2×b×h

Area of triangle =1/2×137×203

Area of triangle =1/2×27811

Area of triangle =13905.5

Therefore, The area of triangle is 13905.5 cm square when height is 137 cm and bae is 203 cm.

To learn more about triangle visit: https://brainly.com/question/2773823

#SPJ9

Find the x-intercept and y-intercept of the line. - 6x + 4y= 15 Write your answers as exact values. Do not write your answers as ordered pairs. x-intercept: 1 Х ? y -intercept: 1

Answers

The equation of a line is line is given as y = mx + c where m is the slope and c is the y-intercept

From the equation

-6x + 4y = 15

Changing into the form of the general equation

4y = 6x + 15

Divide both sides by 4

y = 6y/4 + 15/4

y = 3y/2 + 15/4

the x intercept is 0 while the y intercept is 15/4.

Katie rents a car when spending her vacation in Argentina while she returns the car she has driven 900 miles and used about 36 gallons of gas if you guess cost an average of $4.139 Per gallon estimate how much she spent on fuel

Answers

Given that Katie had driven 900 miles and used about 36 gallons of gas.

The average cost of gas per gallon = $4.139

The amount she spent on gas would be:

[tex]\text{ The average cost of gas per gallon x gallons of gas used}[/tex]

Hence, the amount Katie spent on gas would be:

[tex]\begin{gathered} \text{ }\frac{\text{\$4.139}}{\text{gallon}}\times\text{ 36 gallons} \\ =\text{ \$149.004} \end{gathered}[/tex]

She spent $149.004

CD = 69, BC = 10x + 3. AD = 18x + 44,and AB= 7x- 20. Find BC.

Answers

Answer: A) 83

Explanation:

Representing this segments in a number line, and supposing that they are arranged in alphabetical order:

Here we can see that if we sum all of the segments they must be equal to 18x+44:

[tex]7x-20+10x+3+69=18x+44[/tex]

Combining like terms:

[tex]17x+52=18x+44[/tex]

Now we move all of the terms with x to the right side and all of the independent numbers to the left side:

[tex]\begin{gathered} 52-44=18x-17x \\ 8=x \end{gathered}[/tex]

And now that we know the value of x, we can find BC:

[tex]\begin{gathered} BC=10x+3 \\ BC=10(8)+3 \\ BC=80+3 \\ BC=83 \end{gathered}[/tex]

Which is option A)

19. Which of the following is equal to V-24 ?O-2iV64i 166i-1/2O21 V6

Answers

The given value is,

[tex]\sqrt[]{-24}[/tex]

We can write this as,

[tex]\sqrt[]{-24}=\sqrt[]{24\times-1}[/tex]

As we know, 24 = 4 x 6 and,

[tex]\sqrt[]{-1}=i[/tex]

the above expression can again be rewritten as,

[tex]\sqrt[]{-24\times-1}=\sqrt[]{4\times6}\times i=2i\sqrt[]{6}[/tex]

Thus, the last option is correct.

Given that segment AD is congruent to segment BC, and angle DAB is congruent to CBA; Prove: triangle ABE is isosceles

Answers

Statement | Reason

AD ≅ BC | Given

∠DAB ≅ ∠CBA | Given

AB ≅ AB | Reflexive property of congruence

ΔADB ≅ ABC | SAS postulate

∠DBA ≅ ∠CAB | CPCTC

ΔABE is isosceles | Any triangle with 2 congruent angles is isosceles

The set consisting of all integers between -2 and -1 will be empty

Answers

It is true that the set consisting of all integers between -2 and -1 is empty.

Integers are numbers that are not fraction. They are simply the whole numbers on the number line.

There are positive and negative integers.

Positive Integers are: 1, 2, 3, 4, 5, and so on

Negative Integers are: -1, -2, -3, -4, -5, and so on.

Between -2 and -1, there are no integers. Therefore, the set consisting of all the integers between them is empty.

Can you please help me with this question thank youu.The first one.

Answers

Given that:

- Tara has 18 pairs of white socks.

- She has 12 pairs of colored socks.

You can state the proportion of those types of socks using ratios.

By definition, a ratio can be written in this form:

[tex]a\colon b[/tex]

And it is read "a to b".

In this case, you need to find the ratio of white socks to colored socks, then you have to set up this ratio:

[tex]18\colon12[/tex]

You can simplify the ratio dividing both sides by 6:

[tex]\begin{gathered} 18\div6\colon12\div6 \\ \\ 3\colon2 \end{gathered}[/tex]

Hence, the answer is: Option B.

How many red squares will there be if there are 60 squares?

Answers

• There are 3 red squares.

,

• There are 4 white squares.

To compare them and get the ratio, we can build the following relation:

[tex]\frac{3}{4}=\frac{x}{60}[/tex]

where x is the number of red squares it will be when there are 60 white squares.

Solving for x:

[tex]x=\frac{3}{4}\cdot60[/tex][tex]x=45[/tex]

Answer: C. 45

The slope of the line below is 2 Write the point-slope equation of the line using the coordinates of the labeled point.

Answers

As per given by the question,

there are given that,

The slope of the line is 2, and the point is (3, 10).

Now,

For finding the point slope equation;

From the formula for point slope equation of the line,

[tex]y-y_1=m(x-x_1)[/tex]

Here,

[tex]x_1=3,y_1=10,\text{ and m=2}[/tex]

Then put the value in above formula,

[tex]undefined[/tex]

Hello I am helping my son with independent variable and dependent

Answers

It is given that,

As a plane descends, the more time that passes, the lower the plane's altitude is.

So,

Here, x be the time and y the altitude of the plane.

According to the statement, x be the dependent variable and y be the independent variable.

So, the graph is,

I'll give you the pic.

Answers

Let's use pythagorean theorem to calculate the remaining side:

[tex]\begin{gathered} c=\sqrt[]{a^2+b^2} \\ c=\sqrt[]{7^2+3^2} \\ c=\sqrt[]{21+9} \\ c=\sqrt[]{30} \end{gathered}[/tex]

The area of a square is given by:

[tex]\begin{gathered} A=s^2 \\ \text{Where:} \\ s=\text{One of its sides} \\ A=(\sqrt[]{30})^2 \\ A=30 \end{gathered}[/tex]

From the waiting area, they walked another 0.1 miles to board the plane. The plane left the gate 45 min after they arrived at the waiting area. Part C: what was the length from the waiting area to the airplanes takeoff?

Answers

C) We have to calculate the distance from the waiting area to the plane.

From the waiting area they walked 0.1 miles to board the plane.

Answer: from the waiting area to the plane there is a distance of 0.1 miles.

Find the answers to fill in blank 1. And blank 2.

Answers

EXPLANATION:

We are given the linear equation;

[tex]y-4=3(x+1)[/tex]

To graph this equation, we would begin by re-writing the equation in the slope-intercept form, which is;

[tex]y=mx+b[/tex]

To do this, we first expand the parenthesis;

[tex]y-4=3x+3[/tex]

Next we add 4 to both sides;

[tex]y-4+4=3x+3+4[/tex][tex]y=3x+7[/tex]

We can now begin to plot the various points on the line. Starting from, x = -2 we would have;

[tex]\begin{gathered} x=-2: \\ y=3(-2)+7 \\ y=-6+7 \\ y=1 \end{gathered}[/tex]

We can now go on and plot other points depending on the limit imposed by the graph page.

However, what we have here shows the coordinates from which we may begin;

ANSWER:

[tex]\begin{gathered} (-2,1) \\ That\text{ is;} \\ x=-2,y=1 \end{gathered}[/tex]

To find the length of JK you’d set up and solve:

Answers

According to the statement, to find x, it is necessary to use the following expression:

[tex]7x=3x+14[/tex]

This expression is set up thanks to the definition of a parallelogram. To solve it isolate x to one of the sides of the equation.

[tex]\begin{gathered} 7x=3x+14 \\ 7x-3x=14 \\ 4x=14 \\ x=\frac{14}{4} \\ x=3.5 \end{gathered}[/tex]

x has a value of 14/4 or 3.5.

According to the figure JK measures 7 times x. Use this information to find JK:

[tex]\begin{gathered} JK=7x \\ JK=7(3.5) \\ JK=24.5 \end{gathered}[/tex]

JK measures 24.5.

what is the quotient of {24a^4 b^2 + 36a^2 b-36ab^2 +48 ab}÷(12ab)?

Answers

To divide this polynomial, we will follow steps below:

Step 1

Arrange

step 2

Divide 24a⁴b² by by 12ab

The result will be 2a³b

Write the result at the top of the root sign

Step 3

Mutiply 12ab by 2a³b, the result will be 24a⁴b²

Write the result in the root sign under 24a⁴b²

step 4

subtract, the result is zero

step 5

Take down 36a²b

Step 6

divide 36a²b by 12ab

The result is 3a

write the result at the top of the root sign

step 7

Multiply 12ab by 3a

The result is 36a²b

Write the result in the root sign under 36a²b

Step8

subtract, the result is 0

step 9

Take down -36ab²

step10

Divide -36ab² by 12ab

The result is -3b

Write the result at the top of the root sign

step11

Multiply 12ab by -3b

The result is -36ab²

Write the result in the root sign under -36ab²

step12

subtract, the result is zero

step 13

Take down 48ab

step 14

divide 48ab by 12 ab

The result is 4

write the result at the top root sign

step 15

Mulltiply 12ab by 4

The result is 48ab

Write the result in the root sign under 48 ab and then subtract

The result is zero

Hence the quotient is : 2a³b + 3a -3b + 4

The Quadratic f(x)=x^2-2x-15Using the functions of your graphing calculator calculate the coordinates of the following points (as shown in the calculator videos in this lesson). If the parabola doesn't intersect the x-axis then write "none." If necessary, round to the nearest hundredths place (2 decimal places).a. The vertex using the min/max calculate function.b. X-intercept(s) using the zero calculate function.c. Y-intercept using the value calculate function (w/ a value of x=0).d. Now, copy down the t-table generated by your calculator for integer input values from-3≤x≤3.

Answers

Given: The function below

[tex]f(x)=x^2-2x-15[/tex]

To Determine: The vertex, the x-intercept, the y-intercept, and the table for -3≤x≤3

Solution

The graph of the given function is as shown below

Hence:

The vertex is a minimum value at y = -16 and coordinate (1, -16)

(b) The X-intercepets is x = -3, x = 5, coordinates: (-3, 0) and (5, 0)

(c) The Y-intercept is at y = -15, coordinate: (0, - 15)

(d) The table showing the values of f(x) for -3≤x≤3 is as shown below

which of the following equations represent linear functions? A. x^2+y^2=1 B. x+y=14 C. y=6/x D. y=3(2x+1)

Answers

A linear function has this form:

[tex]y=ax+b[/tex]

Notice that option A cannot be written in this form, because x and y have a square power. If you clear y you'll get:

[tex]y=\sqrt[]{1-x^2}[/tex]

Option B you can write it in the form of a linear equation:

[tex]y=14-x[/tex]

For this option, a = -1 and b = 14

Option C cannot be written in this form:

[tex]y=\frac{6}{x}[/tex]

And option D can be written like that:

[tex]y=6x+3[/tex]

Here, a=6 and b=3.

So, options B and D are linear equations

A pair of shoes is on sale for 20% off. I paid $95. How much were the shoes originally? Write an equation and solve.

Answers

118.75

0.80 * x = 95

x = 95/ 0.80

x = 118.75

With x being the original cost of the shoes.

which fractions represent how to find the probability to rolling a number less than 5 and a number greater than 2?

Answers

[tex]\frac{1}{3}[/tex]

Explanation

The probability of an event is the number of favorable outcomes divided by the total number of outcomes possible

Step 1

find the total of favorable possible

[tex]\begin{gathered} \text{for a dice, } \\ a\text{ number less than 5, it is, 1, 2, 3 or 4, ( 4 favorables outcomes}) \\ a\text{ number greater than 2, it is, 3,4 , 5 or 6} \\ \text{the numbers that have the two options are 3 and 4 ( 2 favorable outcomes)} \end{gathered}[/tex]

favorable outcomes : 2 ( 3 and 4)

Step 2

find the total number of outcomes possilbe

the dice has 6 faces, (numbers, 1, 2, 3, 4, 5 or 6),

possible outcomes : 6 ( 1,2,3,4,5 and 6)

Step 3

finally replace

[tex]\begin{gathered} P=\frac{favorable\text{ outcomes}}{\text{possible outcomes}} \\ P=\frac{2}{6}=\frac{1}{3} \end{gathered}[/tex]

Please Help! Functions and Relations The graph shows the absolute value parent function. which statement best describes the function?

Answers

The function is increasing when, if xa > xb, then f(xa) > f(b).

Let's choose values for x < 0 and for x > 0.

First, let's compare x = -2 and x = -1

-1 > -2

f(-1) < f(-2)

Then, the function is decreasing for x < 0.

Second, let's compare x = 1 and x = 2.

2 > 1

f(2) > f(1)

Then, the function is increasing for x > 0.

Answer: c. The function is increasing when x >0.

question 13Consider the following data: 12, 15, 13, 10, 15, 10. Answer the following questicwrite final answers only. [T/I - 4]#1) What is the mean of the data?#2) What is the median of the data?#3) What is the mode of the data?#4) What is the range of the data?

Answers

Solution:

Given:

The data;

[tex]12,15,13,10,15,10[/tex]

Question 1:

To get the mean:

The mean of a set of numbers, sometimes simply called the average, is the sum of the data divided by the total number of data.

[tex]\begin{gathered} \text{Mean}=\frac{\text{ sum of data}}{n\text{ umber of data}} \\ \text{Mean}=\frac{12+15+13+10+15+10}{6} \\ \text{Mean}=\frac{75}{6} \\ \text{Mean}=12.5 \end{gathered}[/tex]

Therefore, the mean is 12.5

Question 2:

To get the median:

The median of a set of numbers is the middle number in the set (after the numbers have been arranged from least to greatest)

If there is an even number of data, the median is the average of the middle two numbers.

[tex]\begin{gathered} R\text{ earranging the data given in rank order,} \\ 10,10,12,13,15,15 \end{gathered}[/tex]

The data indicates an even number of data. There are 6 numbers in the set.

Hence, the median is the mean of the middle two numbers.

[tex]\begin{gathered} \text{The middle two numbers are;} \\ 12\text{ and 13} \\ \text{Hence, the median is the mean of 12 and 13} \\ \text{Median}=\frac{12+13}{2} \\ \text{Median}=\frac{25}{2} \\ \text{Median}=12.5 \end{gathered}[/tex]

Therefore, the median is 12.5

Question 3:

To find the mode:

The mode of a set of numbers is the number that occurs the most. Hence, the mode of a set of numbers is the number with the highest frequency.

If a set of data has two modes, the data is said to be bimodal.

[tex]\begin{gathered} 10,10,12,13,15,15 \\ \\ \text{From the above, 10 appears twice} \\ 15\text{ also appears twice} \\ \\ \text{Hence, the mode is 10 and 15. The data has two modes, it is a bimodal data.} \end{gathered}[/tex]

Therefore, the modes are 10 and 15.

Question 4:

The range is the difference between the highest and lowest values in a set of numbers.

[tex]\begin{gathered} 10,10,12,13,15,15 \\ \text{Lowest number=10} \\ \text{Highest number=15} \\ \\ \text{Hence, range=highest number-lowest number} \\ \text{Range}=15-10 \\ \text{Range}=5 \end{gathered}[/tex]

Therefore, the range is 5.

Two dice are rolled. What is the probability that the sum of the numbers rolled is either 3 and 8? Express your answer as a fraction in lowest terms or a decimal rounded to the nearest million

Answers

The two dice have 6 numbers each.

Look at the image to find the outcomes:

First, we need to find the total of outcomes with the sum of 3:

T. outcomes with the sum of 3 = 2

Now, find the total of outcomes with the sum of 8 = 5

When we ave in probability the expression "or" we add the probabilities:

In this case,

T. outcomes with the sum of 3 + T. outcomes with the sum of 8.

Replace the values and sum:

P = 2 +5

P = 7

Now, to find the probability divide it by the total of outcomes

Total outcomes= 36. because 6 * 6 = 36

P( sum being 3 or 8) = 7/36

X = y - 4-2x + 3y= 6Solve each system by equation

Answers

Answer: x=-6 and y=-2

Given:

[tex]\begin{gathered} x=y-4 \\ -2x+3y=6 \end{gathered}[/tex]

Having these two equations, we can substitute the first equation with the second equation to solve for y:

[tex]\begin{gathered} -2x+3y=6 \\ \end{gathered}[/tex]

Since the first equation says that x = y - 4,

[tex]\begin{gathered} -2x+3y=6 \\ -2(y-4)+3y=6 \\ -2y+8+3y=6 \\ -2y+3y=6-8 \\ y=-2 \end{gathered}[/tex]

Then, we will substitute this y-value to the first equation to solve for x.

[tex]\begin{gathered} x=y-4 \\ x=-2-4 \\ x=-6 \end{gathered}[/tex]

We now have the values x=-6 and y=-2. To check, let us substitute both values to the second equation

[tex]\begin{gathered} -2x+3y=6 \\ -2(-6)+3(-2)=6 \\ 12-6=6 \\ 6=6 \end{gathered}[/tex]

Therefore, the answer is correct, and the answer is x=-6 and y=-2

The equation of the line of best fit of a scatter plot is y = –7x − 2. What is the the y-intercept?

–7
–2
2
7

Answers

Answer:

The y-intercept of this line is -2.

Can you pls help me with this question thank you

Answers

The Solution:

The difference of c and 7 is either:

[tex]\begin{gathered} c-7\text{ } \\ \text{ or} \\ 7-c \end{gathered}[/tex]

Multiplying the result by 10, we get

[tex]\begin{gathered} 10(c-7) \\ \text{ or} \\ 10(7-c) \end{gathered}[/tex]

We are asked not to simplify any part of the expression.

So, the correct answer is:

[tex]\begin{gathered} 10(c-7) \\ or \\ 10(7-c) \end{gathered}[/tex]

Look at the construction. Which statement is false? XA = YA XP = PY XA = XY

Answers

XA = XY is false. XA and YA are both congruent segments, which means they are equal. The same goes for XP and PY.

WILL GIVE BRAINLIST!!
a bakery. needs to pack 48 donuts, 12 pastries, and 24 cinnamon in identical quantities across all of the boxes. What is the maximum quantity of boxes she can utilize?

Answers

It requires maximum 12 boxes to put 4 donuts, 1 pastry and 2 cinnamon in each box using the Greatest comon factor.

What is Greatest common factor or GCF?

The greatest common factor or GCF is the largest number that can be split into exactly two or more other numbers. It is the "best" thing for reducing the complexity of fractions. A factor is a number that, when multiplied by other numbers, produces the desired numbers in mathematics. Factors are another name for the total that results.

The largest factor that two or more numbers have in common is called the greatest common factor (GCF).

It is given that there are 48 donuts, 12 pastries and 24 cinnamon.

Find the greatest common factor of the given values.

Expand 48,12, and 24 in factors.

48= 2x2x2x2x3

12=2x2x3

24=2x2x2x3

Find the greatest common factors of the three factored-out numbers.

GCF=2x2x3=12

So, it requires maximum 12 boxes to put 4 donuts, 1 pastry and 2 cinnamon in each box using the GCF.

To know more about the greatest common factor:

https://brainly.com/question/219464

#SPJ1

Other Questions
7. Tres hermanos tienen una bolsa de golosinas. Quieren regalarle a su padre un trozo de twizzler cada uno. Si cada trozo de twizzler que regalan tienen 100 mm, cuntos centmetros de twizzler tendr en total el padre? Particle Mass (amu)proton 1.007593neutron 1.008982deuterium 2.014186tritium 3.016448helium 3 3.015779helium 4 4.0038731.0 ev = 1.6 10-19 j1.0 amu = 931 Mev1 angstrom () = 1 X 10-10mh = 6.63 10-34 J-s.Always use significant figure rules. Remember that these rules apply to all numbers that are measurements.The mass of a deuterium nucleus 21H is less than its components masses. Calculate the binding energy. Mev What is the slope of a line that is parallel to the line y = 3/4x + 2?-4/3-3/43/44/3 john's neighbor theresa informs him that her employer plans to go public. john wants to buy 1,000 shares of the new company's stock at the initial public offering. john calls his broker and receives a 200-page document that has information about the company and the proposed initial offering, but it does not include the date and price of the initial offering. john has received Find the value of 441 + 16 + 4 . Julia is buying vegetables at afarmer's market. One of the stands isselling zucchini for $1.25 per pound.The total weight of the zucchini shehas selected is 3.8 pounds. Howmuch will she pay for hello I seem to be having issues on this problem Use the point-slope formula to write an equation of the line that passes through (-3, 2) and (-6, -2).Write the answer in slope-intercept form (if possible). Billy is solving the inequality 4x - 4 > 20:His work is shown:4x - 4 > 20+4+44x > 244 4X < 6Did Billy solve the problem correctly? If not, where did Billy make a mistake A certain drug is made from only two ingredients: compound A and compound B. There are 5 milliliters of compound A used for every 7 milliliters of compound B. If a chemist wants to make 1056 milliliters of the drug, how many milliliters of compound A are needed?what is the answer? A post made to your favorite social media application 5 years ago can add information to your digital footprint. true false your teacher has selected a program for your senior high school, write a letter to him explaining why you prefer different one. How do I write the addresses Find the values of x and y to make the following numbers equal There are 48 employees in a company. On a certain day, 36 were present. What percent showed up for work' Logan loves to order beef and broccoli from his local takeout restaurant. His doctor tells him that usually takeout meals like that contain more sodium than he needs during the day. Without giving up his favorite dish, what is the BEST way for Logan to reduce his salt intake? Write the factored form of the least common denominator needed to simplify this expression.9419 + 3+g2 + 29- 15+5 Peter has $ 40 to spend on rides at the state fair. Each ride costs $3. Find the recursiveand the explicit function.Recursive Function:Explicit Function: (-9,-5),(-6,1) and (5,8)What is the perimeter? Write the coordinate point for the vertex of this parabola: 1,2 x= 4.* 4. Students were asked how many hours a weeknight they spend on homework and exercise. The dot plots display the results. Which statement is not supported? Homework Exercise LG 2 3 4 5 Number of Hours 1 2 3 4 5 Number of Hours A. More time is spent on homework than exercise. B. The mode activity times are equal. C. The median exercise time is not equal to the median homework time. D. The range of homework hours is greater for homework than for exercise